How can mathematical induction be used to prove the triangle inequality?

Click For Summary
SUMMARY

The discussion centers on using mathematical induction to prove the triangle inequality. Participants confirm the validity of the approach by expressing the inequality as |a_1 + a_2 + ... + a_k + a_{k+1}| = |(a_1 + a_2 + ... + a_k) + a_{k+1}|. The method involves applying the induction hypothesis effectively after utilizing a provided hint. This structured approach simplifies the proof process, demonstrating the power of mathematical induction in validating the triangle inequality.

PREREQUISITES
  • Understanding of mathematical induction
  • Familiarity with the triangle inequality theorem
  • Basic knowledge of absolute values
  • Experience with algebraic manipulation
NEXT STEPS
  • Study the principles of mathematical induction in depth
  • Explore proofs of the triangle inequality in various contexts
  • Learn about the properties of absolute values in mathematics
  • Practice algebraic manipulation techniques for complex proofs
USEFUL FOR

Mathematics students, educators, and anyone interested in advanced mathematical proofs, particularly those focusing on inequalities and induction techniques.

pzzldstudent
Messages
43
Reaction score
0
My professor said this was the triangle inequality. We're to use mathematical induction to prove it. I've gotten some work done, and after "proving" it, it just seems to easy. :|

http://answerboard.cramster.com/advanced-math-topic-5-321495-0.aspx" .
 
Last edited by a moderator:
Physics news on Phys.org
The work you've done is essentially correct - the idea is to write

<br /> |a_1 + a_2 + \dots + a_k + a_{k+1}| = |(a_1 + a_2 + \dots + a_k) + a_{k+1}|<br />

then use the hint once, then your induction hypothesis.
 
thank you

thanks!
 
Question: A clock's minute hand has length 4 and its hour hand has length 3. What is the distance between the tips at the moment when it is increasing most rapidly?(Putnam Exam Question) Answer: Making assumption that both the hands moves at constant angular velocities, the answer is ## \sqrt{7} .## But don't you think this assumption is somewhat doubtful and wrong?

Similar threads

Replies
7
Views
3K
Replies
9
Views
6K
Replies
6
Views
2K
  • · Replies 4 ·
Replies
4
Views
5K
  • · Replies 6 ·
Replies
6
Views
3K
  • · Replies 4 ·
Replies
4
Views
2K
  • · Replies 1 ·
Replies
1
Views
7K
Replies
11
Views
2K
  • · Replies 8 ·
Replies
8
Views
3K
  • · Replies 10 ·
Replies
10
Views
3K